Đến nội dung

tuananh2000 nội dung

Có 216 mục bởi tuananh2000 (Tìm giới hạn từ 02-06-2020)



Sắp theo                Sắp xếp  

#569542 Chứng minh rằng : $\prod (a+b-c)^{2}\geq \prod...

Đã gửi bởi tuananh2000 on 02-07-2015 - 22:11 trong Bất đẳng thức và cực trị

Cho $a;b;c$ là các số thực tùy ý . Chứng minh rằng :

$\prod (a+b-c)^{2}\geq \prod (a^{2}+b^{2}-c^{2})$

 

 




#569541 $for$ i:=1 $to$ n $do$

Đã gửi bởi tuananh2000 on 02-07-2015 - 22:07 trong Góc Tin học

Phân tích thời gian thực hiện của các đoạn chương trình ( Mấy bác nói cụ thể nhá )

1. $for$ i:=1 $to$ n $do$

     $if$ i mod 2=0 $then$ c:=c+1;

2. a:=0

    b:=0

    c:=0

$for$ i:=1 $to$ n $do$

    $begin$

a:=a+1

b:=b+i

c:=c+1*i;

   $end;$

3. d:=0

$while$ n>0 $do$

$begin$

      n:=n div 2;

      d:=d+1;

$end;$

P/s: Có ai giúp mình phần này với , tân binh không hiểu mô tê gì hết  :wacko:  :wacko:




#569441 Topic tổng hợp một số bất đẳng thức trong kì thi MO các nước

Đã gửi bởi tuananh2000 on 02-07-2015 - 14:23 trong Bất đẳng thức - Cực trị

Thế thì $abc=1$ à

Mình cũng chẳng biết @@




#569420 Topic tổng hợp một số bất đẳng thức trong kì thi MO các nước

Đã gửi bởi tuananh2000 on 02-07-2015 - 10:59 trong Bất đẳng thức - Cực trị

Bài 165 (Mediterranean Mathematical Competition 2009).Chứng minh rằng với mọi  $a,b,c $ dương ta luôn có:

$$\sum \frac{ab}{a^2+ab+b^2}\leq \frac{a}{2a+b}+\frac{b}{2b+c}+\frac{c}{2c+a}$$

Đặt $(a;b;c)=(\frac{x}{y};\frac{y}{z};\frac{z}{x})$ , đưa bđt cần cm thành

$\sum \frac{x^{2}}{x^{3}+x^{2}+1}\leq\sum \frac{x}{2x+1}$

Hay $\sum(x-1)^{2}(x+1)x\geq 0$  (đúng)

Vậy bất đẳng thức đc cm

Dấu đẳng thức xảy ra khi $a=b=c=1$ 




#569223 b) Chứng minh rằng: $\sqrt{a}+\sqrt{b}+...

Đã gửi bởi tuananh2000 on 01-07-2015 - 09:24 trong Bất đẳng thức và cực trị

e chưa hiểu cái chỗ đó lắm ạ

Đó là bđt quen thuộc kiểu $(\sum x)^{2}\geq 3xy$ thì thay $x=ab$ rồi căn bậc 2 nữa là được




#569222 Tìm Min P= a+b+c

Đã gửi bởi tuananh2000 on 01-07-2015 - 09:21 trong Bất đẳng thức và cực trị

Cho a, b, c là các số thực thỏa mãn $a\geq 4, b\geq 5, c\geq 6$ và $a^{2}+b^{2}+c^{2}\doteq 90$.

Tìm Min P= a+b+c

Mượn điểm rơi của anh Bad Boy

Ta đoán được điểm rơi xảy ra khi $a=4;b=5;c=7$

Khi đó $GTNN$ của $P=16$

Ta cm $P\geq 16$ bằng pp phản chứng và một ít $Abel$

Giả sử $a+b+c< 16$ thì $b+c = 16-a\geq 12$ và $c=16-a-b\geq 7$

Ta có $4a+5b+7c=c.7+b.5+a.4=c.2+(b+c)+4(\sum a)> 2.7+12+4.16=90$

Theo $B.C.S$ thì $(4^{2}+5^{2}+7^{2})(a^{2}+b^{2}+c^{2})\geq (4a+5b+7c)^{2}> 90^{2}$

Hay $\sum a^{2}> 90$ $( vô lí)$

Từ đó ....

P/s: Bài này hơi quen :icon6:  :icon6: 




#569217 b) Chứng minh rằng: $\sqrt{a}+\sqrt{b}+...

Đã gửi bởi tuananh2000 on 01-07-2015 - 08:58 trong Bất đẳng thức và cực trị

cho các số duơng a,b,c thỏa mãn a+b+c=3
 

b) Chứng minh rằng: $\sqrt{a}+\sqrt{b}+\sqrt{c}\geq ab+bc+ca$

$AM-GM$ thì $\sqrt{a}+\sqrt{a}+a^{2}\geq 3a$ suy ra $2\sum \sqrt{a}+\sum a^{2}\geq 3\sum a=(\sum a)^{2}$ suy ra $đpcm$




#569008 $0,67875_{10}=0,101011011..._{2}$

Đã gửi bởi tuananh2000 on 29-06-2015 - 23:32 trong Số học

Mình không hiểu $0,67875_{10}=0,101011011..._{2}$ lắm , mấy bạn giải thích kĩ một tí và đưa ra vài vd với ạ :namtay




#568993 $\frac{a^5-a^2}{a^5+b^2+c^2}+\frac{b^...

Đã gửi bởi tuananh2000 on 29-06-2015 - 21:43 trong Bất đẳng thức và cực trị

Nếu điều kiện là $a^2+b^2+c^2=3$ thì khá dễ bằng HFT + D.A.C., nếu là $a^2+b^2+c^2\geqslant 3$ thì anh Cẩn giải như sau:

Bất đẳng thức tương đương với: $\sum \dfrac{1}{a^5+b^2+c^2}\leqslant \dfrac{3}{a^2+b^2+c^2}$

Đến đây ta suy ra chỉ cần chứng minh bất đẳng thức khi $a^2+b^2+c^2=3$, anh chị nào giải thích giúp em tại sao có được nó.

Mình tham khảo được tài liệu, chắc của anh Cẩn thì cm như sau 

Untitled6.png

Untitled7.png




#568989 Tìm số nguyên k thỏa mãn$\frac{8k^2-25}{5+3k}...

Đã gửi bởi tuananh2000 on 29-06-2015 - 21:33 trong Số học

Tìm số nguyên k thỏa mãn$\frac{8k^2-25}{5+3k}$ là số nguyên

Bạn nhân thêm$9$ vào p/s rồi lấy tử chia mẫu giải bình thường , sau đó thử lại và lấy giá trị thỏa mãn




#568236 Chứng minh: $S\leq \frac{1}{4}\sqrt...

Đã gửi bởi tuananh2000 on 26-06-2015 - 08:23 trong Bất đẳng thức và cực trị

 Cho a, b, c là ba cạnh của một tam giác và S là diện tích của tam giác đó. Chứng minh:

$S\leq \frac{1}{4}\sqrt{a^{4}+b^{4}+c^{4}}$

Theo công thức Hê-rông thì $S=\sqrt{p(p-a)(p-b)(p-c)}$ với $p=\frac{\sum a}{2}$

Ta có bđt cần cm tương đương với 

$(\sum a)\prod (a+b-c)\leq \sum a^{4}$

Mặt khác $VT\leq \prod a(\sum a)\leq \sum a^{2}b^{2}\leq \sum a^{4}=VT$

Dấu đẳng thức xảy ra khi tam giác đều 




#568234 sách hay

Đã gửi bởi tuananh2000 on 26-06-2015 - 08:14 trong Kinh nghiệm học toán

Cho em hỏi học tin học thì nên mua những quyển nào ( chuyên sâu luôn ấy ) và cách học hiệu quả ra sao ạ? :icon6:  :icon6:




#568140 $$\frac{1}{11+a^2}+\frac{1}...

Đã gửi bởi tuananh2000 on 25-06-2015 - 18:27 trong Bất đẳng thức - Cực trị

Sr bạn dog , mình không để ý do thấy cách bạn nhiều $TH$ quá  :luoi:  :luoi:




#568139 Giải phương trình $\sqrt{x-1}+\sqrt{x-3}+2...

Đã gửi bởi tuananh2000 on 25-06-2015 - 18:25 trong Phương trình, hệ phương trình và bất phương trình

Giải phương trình: $\sqrt{x-1}+\sqrt{x-3}+2\sqrt{(x-1)(x^{2}-3x+5)}=4-2x$

Bạn xem lại đề đi nhé với đk $x\geq 3$ thì $VT$ dương còn $VP$ âm rồi nên $PTVN$




#568137 $$\frac{1}{11+a^2}+\frac{1}...

Đã gửi bởi tuananh2000 on 25-06-2015 - 18:10 trong Bất đẳng thức - Cực trị

Từ từ mà lần thôi  :icon6:  :icon6:  :icon6:

Ta cm bđt sau $\frac{1}{11+a^{2}}\leq \frac{1}{12}+\frac{a-1}{72}$ hay $\frac{(a-1)^{2}(a-5)}{6(132+a^{2})}\leq 0$ ( luôn đúng với $0\leq a\leq 4$

TT rồi cộng theo vế được $ĐPCM$

Dấu đẳng thức xảy ra khi $a=b=c=d=1$




#567827 Topic tổng hợp một số bất đẳng thức trong kì thi MO các nước

Đã gửi bởi tuananh2000 on 24-06-2015 - 14:07 trong Bất đẳng thức - Cực trị

Bài 151(Romania TST): Cho a,b,c>0. CMR: nếu $a^2+b^2+c^2=3$ thì

$\sum \frac{1}{a^2+b^3+c^2}\leq 1$

 

Ta xét $\frac{1}{a^{2}+b^{3}+c^{2}}=\frac{1}{b^{3}-b^{2}+3}$

Ta cm$\frac{1}{b^{3}-b^{2}+3}\leq \frac{1}{3}+\frac{b^{2}-1}{18}$ hay $(b-1)^{2}\left [ \frac{b(6-b^{2})+3-b^{2})}{18(b^{3}+3-b^{2})} \right ]$

Bất đẳng thức trên hiển nhiên đúng vì $b>0$ và $b^{2}\leq 3$

TT rồi cộng theo vế được $ĐPCM$

Dấu $'='$ xảy ra khi $a=b=c=1$




#567795 $\begin{cases} a-b+c-d=2 \\ a^2-b^2+c^2-d^2=6...

Đã gửi bởi tuananh2000 on 24-06-2015 - 11:02 trong Phương trình - Hệ phương trình - Bất phương trình

 Giải ra rồi, bữa nhân nhẩm ngu nên ........

 Đặt $\left ( a+c~,ac~,b+d~,bd \right )=(m,n,p,q)$ thì viết hệ lại dưới dạng

$$\left\{\begin{matrix} m-p=2\\ m^2-2n-p^2+2q=6\\ m^3-3mn-p^3+3pq=20\\ m^4-4m^2n+2n^2-p^4+4p^2q-2q^2=66 \end{matrix}\right.$$

 Ta có : $m^2-2n-p^2+2q=6\Leftrightarrow (p+2)^2-2n+2q-p^2=6\Leftrightarrow 2p-n+q=1\Leftrightarrow n=2p+q-1$

 Thay $m=p+2$ và $n=2p+q-1$ vào phương trình (3) và thu gọn ta được $p=2q+2$

 Thay $m=2q+4~;n=5q+3~;p=2q+2$ vào phương trình (4) ta thu được : $q=0\Rightarrow m=4;n=3;p=2$

 Với $m=4;n=3$ suy ra $(a,c)=(1,3)$ và hoán vị

 Với $p=2;n=0$ suy ra $(b,d)=(2,0)$ và hoán vị

 Vậy nghiệm của hệ là $(a,b,c,d)\in \left \{ (1,2,3,0);(1,0,3,2);(3,1,2,0);(3,1,0,2) \right \}$

Ráp vào pt bậc 4 hơi gian nan  :icon6:  :icon6: , có cách nào khác không bạn ?




#567792 Tính $A=\frac{1}{2+a}+\frac{1}...

Đã gửi bởi tuananh2000 on 24-06-2015 - 10:53 trong Đại số

 

 

Cho $x^{2}+y^{2}+z^{2}= 1$ và $x^{3}+y^{3}+z^{3}= 1$ .Tính B=$xyz$

Ta có $x^{2}+y^{2}+z^{2}=1$ nên $0\leq x^{2}\leq 1$ hay $-1\leq x\leq 1$

Theo gt $x^{2}+y^{2}+z^{2}=x^{3}+y^{3}+z^{3}= 1$ nên $\sum x^{2}(1-x)=0$

Mặt khác $x^{2}\geq 0$ và $1-x\geq 0$ với $-1\leq x\leq 1$ nên$\sum x^{2}(1-x)\geq 0$

Dấu bằng xảy ra khi có ít nhất một số bằng $0$ nên $B=0$




#567780 Topic tổng hợp một số bất đẳng thức trong kì thi MO các nước

Đã gửi bởi tuananh2000 on 24-06-2015 - 09:49 trong Bất đẳng thức - Cực trị

Bài 145(China TST): Cho a,b,c >0. CMR: nếu a+b+c=3

$\frac{1}{a^2+b+c}+\frac{1}{b^2+c+a}+\frac{1}{c^2+a+b}\leq 1$

Ta xét $\frac{1}{a^{2}+b+c}=\frac{1}{a^{2}-a+3}$

Ta cm$\frac{1}{a^{2}-a+3}\leq \frac{1}{3}+\frac{a-1}{9}$ hay $\frac{(a-1)^{2}(a-3)}{a^{2}-a+3}\leq 0$ ( luôn đúng với $0\leq a\leq 3$)

TT với các hạng tử còn lại rồi cộng theo vế có $ĐPCM$

Dấu $'='$ xảy ra khi $a=b=c=1$




#567772 Topic tổng hợp một số bất đẳng thức trong kì thi MO các nước

Đã gửi bởi tuananh2000 on 24-06-2015 - 09:10 trong Bất đẳng thức - Cực trị

 

 

Bài 147 (Làm mạnh bài USAMO 2003). Cho ba số $a,\,b,\,c$ dương. Chứng minh rằng

\[\sum \frac{(2a+b+c)^{2}}{2a^{2}+(b+c)^{2}}+\frac{3(a^2+b^2+c^2)}{(a+b+c)^2} \leqslant 9..\]
 

Chuẩn hóa $\sum a=3$

Ta có $\frac{(2a+b+c)^{2}}{2a^{2}+(b+c)^{2}}+\frac{3a^{2}}{(\sum a)^{2}}=\frac{(a+3)^{2}}{3a^{2}-6a+9}+\frac{3a^{2}}{9}$

Ta cm $\frac{(a+3)^{2}}{3a^{2}-6a+9}+\frac{3a^{2}}{9}\leq 3+2(a-1)$ hay $(a-1)^{2}(a-6)a\leq 0$ ( luôn đúng với $0\leq a\leq 3$)

Cộng các vế với nhau ta đc $ĐPCM$

Dấu $'='$ xảy ra khi $a=b=c>0$




#567578 Chứng minh rằng :c=$\sqrt[3]{2}+\sqrt[3]{4...

Đã gửi bởi tuananh2000 on 23-06-2015 - 08:47 trong Đại số

Khẳng định của bạn lấy ở đâu vậy, toán học không có  :(

Số vô tỉ cộng với một số vô tỉ hoặc số vô tỉ cộng với một số hữu tỉ đều được số vô tỉ mà




#567569 Chứng minh rằng :c=$\sqrt[3]{2}+\sqrt[3]{4...

Đã gửi bởi tuananh2000 on 23-06-2015 - 08:01 trong Đại số

Đặt  $\sqrt[3]{2}=x$.  $x$ là số vô tỉ

       $c=x+x^2$ 

Giả sử  $c$  là số hữu tỉ thì  $x^2+x+1$  là số hữu tỉ

Do  $x>1$,  $x-1$  là số vô tỉ nên 

     $(x-1)(x^2+x+1)$  là số vô tỉ   $\leftrightarrow x^3-1$   là số vô tỉ   $\leftrightarrow  1$   là số vô tỉ  (vô lí)

Vậy  $c$  là số vô tỉ  

Nếu bạn suy được $\sqrt[3]{2}$ là số vô tỉ thì bình phương của nó là số vô tỉ hoặc hữu tỉ . Khi đó $c$ chắc chắn là số vô tỉ rồi  :icon6:  :icon6:




#567334 Tìm $GTNN$ của $P=x^{2}+my^{2}+nz^{2...

Đã gửi bởi tuananh2000 on 21-06-2015 - 20:39 trong Bất đẳng thức và cực trị

Trong quyển tuyển tập 5 năm kỷ yếu của GGTH mình đã giải quyết bài toán này.

 

 

Lời giải. Phần (a) khá đơn giản, bạn đọc có thể tự chứng minh lấy bằng cách sử dụng đạo hàm. Ở đây ta chỉ quan tâm đến phần (b) của bài toán.

Giả sử $P$ đạt giá trị nhỏ nhất bằng $2t \;(t>0).$ Khi đó ta có bất đẳng thức sau

$$x^2+ay^2+bz^2 \ge 2t,$$

tương đương với

$$x^2+ay^2+bz^2 \ge 2t(xy+yz+zx). \quad (5.1)$$

 

hay

$$x^2+ay^2+bz^2+t(x^2+y^2+z^2) \ge 2t(xy+yz+zx)+t(x^2+y^2+z^2),$$

$$(t+1)x^2+(t+a)y^2+(t+b)z^2\ge t(x+y+z)^2.$$

Sử dụng bất đẳng thức Cauchy-Schwarz, ta có

$$(t+1)x^2+(t+a)y^2+(t+b)z^2= \frac{x^2}{\frac{1}{t+1}}+\frac{y^2}{\frac{1}{t+a}}+\frac{z^2}{\frac{1}{t+b}}\ge \frac{(x+y+z)^2}{\frac{1}{t+1}+\frac{1}{t+a}+\frac{1}{t+b}}.$$

Như vậy, ta cần cần chọn $t$ sao cho

$$\frac{1}{t+1}+\frac{1}{t+a}+\frac{1}{t+b}=\frac{1}{t},$$

quy đồng và thu gọn, ta được $$2t^3+(1+a+b)t^2-ab=0.$$ Kết hợp với câu $(a)$ ta suy ra $P$ đạt giá trị nhỏ nhất là $2t$ với $t$ là nghiệm dương duy nhất của đa thức $$f(t)=2t^3+(1+a+b)t^2-ab.$$ Đẳng thức xảy ra khi và chỉ khi $(t+1)x=(t+a)y=(t+b)z.$

Em mới THCS nên anh giải câu a) với được không ạ? :icon6:




#567322 Đề thi tuyển sinh vào lớp 10 chuyên Toán tỉnh Quảng Bình

Đã gửi bởi tuananh2000 on 21-06-2015 - 18:23 trong Tài liệu - Đề thi

 

ĐỀ THI TUYỂN SINH VÀO LỚP 10 CHUYÊN TOÁN TỈNH QUẢNG BÌNH

 

 

 Câu 1 ( 2,0 điểm ) Cho biểu thức

$$P=\left ( \frac{4\sqrt{x}}{2-\sqrt{x}}-\frac{8x}{4-x} \right ):\left ( \frac{\sqrt{x}-4}{x+2\sqrt{x}}+\frac{1}{\sqrt{x}} \right )$$

  với $x>0,x\neq 1,x\neq 4$

 a) Rút gọn P

 b) Tìm$x$ để $P=-1$

 Câu 2 ( 2,5 điểm )

 a) Giải phương trình $x^2+x-4\sqrt{3x+1}+6=0$

 b) Trong hệ toạ độ Oxy, cho Parabol $(P):~y=x^2$ và đường thẳng $(d):~y=2mx+2$ ( $m$ là tham số ). Tìm $m$ để $(d)$ cắt $(P)$ tại 2 điểm phân biệt A và B sao cho $S_{OAB}=2\sqrt6$

 Câu 3 ( 1,0 điểm )

    Cho các số thực dương $a,b,c$ thoả mãn $ab+bc+ca=11$. Tìm GTNN

$$P=\frac{5a+5b+2c}{\sqrt{12(a^2+11)}+\sqrt{12(b^2+11)}+\sqrt{c^2+11}}$$ 

 Câu 4 ( 1,0 điểm ) Tìm số tự nhiên $n$ biết $n+S(n)=2015$

 Câu 5 ( 3,5 điểm )

   Cho tam giác ABC nhọn nội tiếp (O), ba đường cao AD,BE,CF cắt nhau ở H và cắt (O) tại M,N,P

   a) Chứng minh M đối xứng H qua BC

   b) Chứng minh $(AHB)=(BHC)=(CHA)$

   c) Tính $T=\frac{AM}{AD}+\frac{BN}{BE}+\frac{CP}{CF}$

 

Câu 2)
a) $Pt \Leftrightarrow (x-1)^{2}+(\sqrt{3x+1}-2)^{2}=0$ suy ra $x=1$

b)Ta có pt hoàng độ giao điểm của $d$ và $(P)$ như sau $x^{2}-2mx-2=0$

Xét $\Delta =m^{2}+2> 0$ nên pt luôn có $2$ nghiệm phân biệt

Theo $Viet$ có $\left\{\begin{matrix} x_{1}+x_{2}=2m & \\ x_{1}.x_{2}=-2 & \end{matrix}\right.$

Mà $AB^{2}=(x_{1}-x_{2})^{2}+(x_{1}^{2}-x_{2}^{2})^{2}=\left [ (x_{1}+x_{2})^{2}-4x_{1}x_{2}) \right ]\left [ 1+(x_{1}+x_{2})^{2} \right ]=(4m^{2}+8)(4m^{2}+1)$

Theo công thức tính độ dài một điểm tới đoạn thẳng thì

$h=d(O;(d))=\frac{\left | 0-0+2 \right |}{\sqrt{4m^{2}+1}}$

Mặt khác $AB^{2}=\frac{4S^{2}}{h^{2}}=24(4m^{2}+1)$

Hay $(4m^{2}+8)(4m^{2}+1)=24(4m^{2}+1)$

Dễ có $ m=-2$ hoặc $m=2$




#567273 Tìm $GTNN$ của $P=x^{2}+my^{2}+nz^{2...

Đã gửi bởi tuananh2000 on 21-06-2015 - 11:02 trong Bất đẳng thức và cực trị

1 Cho $m;n$ là các hằng số dương và $x;y;z$ là các số thực thay đổi sao cho $xy+yz+zx=1$.Tìm $GTNN$ của $P=x^{2}+my^{2}+nz^{2}$

2 Cho $a,b,c$ là các hằng số dương và $x,y,z$ là những số dương thay đổi sao cho $ax+by+cz=xyz$. Tìm $GTNN$ của $P=x+y+z$